3
$\begingroup$

What can be said about the Hausdorff dimension of the image of a set by a $W^{1,1}$ map?

In other words, what is the relationship between $\mathrm{dim}_H f(A)$ and $\mathrm{dim}_H A$, where $f \in W^{1,1}$?

Does the result also hold if $f$ is a $BV$ function?

$\endgroup$
4
  • 3
    $\begingroup$ To avoid problems with null sets, do you want to take the essential image? $\endgroup$ Mar 30, 2019 at 3:36
  • $\begingroup$ @NateEldredge Yes. $\endgroup$
    – Riku
    Mar 30, 2019 at 10:00
  • $\begingroup$ Could you please clarify what are the domain and range of $f$? In my answer it is assumed that $f\colon[0,1]\to[0,1]$, but maybe you want something different. $\endgroup$
    – Skeeve
    Mar 30, 2019 at 14:35
  • $\begingroup$ @Skeeve That's alright. $\endgroup$
    – Riku
    Mar 30, 2019 at 17:00

1 Answer 1

2
$\begingroup$

If $f$ is Lipschitz then $\dim_H f(A) \le \dim_H A$, since $H^s_\delta(f(A)) \le Lip(f) \cdot H^s_\delta(A)$ for any $\delta>0$.

However there exists an absolutely continuous function $f\in W^{1,1}$ such that $\dim_H(f(A)) > \dim_H A$ for some set $A$. Let us construct such a function (by a modification of the classical Cantor function).

Consider the classical one-third Cantor set $T\subset [0,1]$ and one-fifth Cantor set $F \subset [0,1]$. Let $T_n$ and $F_n$ denote respectively the unions of $2^n$ closed segments of lengths $3^{-n}$ and $5^{-n}$ such that $T = \bigcap_{n\in \mathbb N} T_n$ and $F = \bigcap_{n\in\mathbb N} F_n$. The standard computation shows that $\dim_H T = \frac{\ln 2}{\ln 3} > \frac{\ln 2}{\ln 5} = \dim_H F$, so we are going to define $f$ in such a way that $f(F) = T$.

Now let $f_n$ denote the increasing function mapping the endpoints of $F_n$ to the endpoints of $T_n$, interpolated linearly for the rest of $x\in[0,1]$. Arguing as in the construction of the classical Cantor function we get that $f_n$ converges uniformly to a continuous function $f$. Moreover $f_n'$ converges pointwise on the open set $F^c$ (hence a.e.).

Let $g(x):= \sup_{n\in\mathbb N}|f_n'(x)|$ for a.e. $x$. It is possible to show that $$ \|g\|_1 \le 1 + \sum_{n=1}^\infty 2^n \cdot 5^{-n} \cdot \frac{3^{-n}}{5^{-n}} < \infty, $$ hence by dominated convergence $f'\in L^1[0,1]$, i.e. $f$ is absolutely continuous (and $f\in W^{1,1}$). Since $f(F_n) = T_n$ for all $n$ it follows that $f(F) = f(T)$.

$\endgroup$
9
  • $\begingroup$ Thank you. That's very useful. Do you happen to know any result on the Hausdorff dimension of the graph of a $W^{1,1}$ or $BV$ function? for Lipschitz maps, we have that it is equal to $1$. Does the same hold for $W^{1,1}$ or $BV$ functions? $\endgroup$
    – Riku
    Mar 30, 2019 at 12:28
  • $\begingroup$ Also, why is the dominated convergence theorem relevant? Do you know anything about the pointwise convergence of $f'_n$? $\endgroup$
    – Riku
    Mar 30, 2019 at 12:36
  • $\begingroup$ $f_n'$ converges pointwise on the open set $F^c$. In fact since $F^c = \bigcup_{n\in\mathbb N} F_n^c$ and $F_n^c \subset F_{n+1}^c$ are all open, for any $x\in F^c$ there exists $n\in \mathbb N$ and an open neighbourhood of $x$ on which $f_k$ is stationary for $k\ge n$, hence also $f_k'$ is stationary. $\endgroup$
    – Skeeve
    Mar 30, 2019 at 13:10
  • $\begingroup$ Thank you. What about the Hausdorff dimension of the graph of a $W^{1,1}$ or $BV$ function? Can we say that there are functions such that it is strictly greater than 1? $\endgroup$
    – Riku
    Mar 30, 2019 at 14:12
  • $\begingroup$ By graph do you mean the image, i.e. $f(\mathbb R)$, or really the graph, i.e. the set of points $(x,f(x))$ where $x\in \mathbb R$? $\endgroup$
    – Skeeve
    Mar 30, 2019 at 14:20

Your Answer

By clicking “Post Your Answer”, you agree to our terms of service and acknowledge you have read our privacy policy.

Not the answer you're looking for? Browse other questions tagged or ask your own question.